IPAP Exam Master (Belvoir) - Pulmonology

Ace your homework & exams now with Quizwiz!

A 60-year-old man presents with a 1-year history of increasing dyspnea. The patient used to work as a construction worker, and he is a chronic smoker. A chest radiograph shows diaphragmatic pleural plaques and interstitial lung disease. What is the most likely diagnosis?

Asbestosis

A 72-year-old man presents with progressive shortness of breath over the years. He denies chest pain or a history of smoking. The patient was in the construction business for many years, and before that he worked as a ship builder. Chest x-ray reveals marked interstitial fibrosis and calcified pleural plaques on the lateral chest wall. What is the most likely diagnosis?

Asbestosis

A 5-year-old boy presents with an erythematous skin rash. It is associated with intense itching. The boy's mother has noticed that her son's rash has been recurrent, with 3 to 4 episodes per year. The itching and rash increases after consumption of certain foods. On examination, erythematous raised papules are seen on the cheek, trunks, and upper arms; an example is shown (atopic dermatitis). What is most likely associated with the boy's condition?

Asthma

A 7-year-old boy presents with a 1-week history of wheezing and dyspnea on any exertion (with productive cough). On physical examination, bilateral rhonchi are heard. After a few days of treatment, spirometry is done on the patient. The findings are shown in the table (reduction in both forced vital capacity (FVC) and forced expiratory volume in 1 second (FEV1), but FEV1 is more affected than FVC. Therefore, there is a decrease in the FEV1/FVC ratio below the predicted levels.). Total lung capacity (TLC) is 111% on spirometry. The TLC is normal (80-120%) What is the most likely diagnosis?

Asthma

You are evaluating a 78-year-old man who lives in a nursing home due to moderate Parkinsonism. The patient does not have any specific complaints. However, nursing home staff have noticed that he does not seem as alert as usual and is spending more time in his room sleeping. He is also eating less and has dropped 2 lb in the last 2 weeks. The patient has the following exam: Vital signs: temp 100.8° F; pulse: 100 bpm; respiration: 25/min and bp: 120/70 mm Hg. On exam, he is aware of the place, but not the exact time or date. His HEENT exam is unremarkable. His heart exam reveals tachycardia, but no other abnormality. His lung sounds are decreased at the bases. Based on this presentation, what should be included as part of the initial workup?

Complete blood count

A 52-year-old woman with a history of untreated bronchogenic carcinoma presents with a 1-week history of dyspnea, facial edema, and marked neck vein distention; the distention has progressed over the past 36 hours. A chest X-ray confirms a right hilar mass with a small pleural effusion in the right hemithorax. What step is most appropriate?

Consult an oncologist for radiation therapy

A 4-year-old girl has always been below 3% for her height and weight. Her mother says she eats 3 meals a day with healthy snacks. The family tries to maintain a low-fat, high-fiber diet. Her parents and sisters are of above average height and average weight. Other than occasional upper respiratory infections and 1 episode of pneumonia last winter, the girl has been fairly healthy. Her failure to thrive workup is normal, other than a sweat test, which reveals a high chloride concentration. What is an important modification to her diet as a result of the most likely diagnosis?

Fat-soluble vitamin supplements

An 30-year-old immigrant worker presents with a chronic cough, blood-stained sputum, and night sweats. Physical examination reveals a wasted middle-aged man with bronchial breathings on the right upper lobe. His sputum examination under microscope shows acid-fast staining rods, and his PPD is 15 mm. Based on clinical findings, sputum examination, and PPD result, pulmonary tuberculosis is diagnosed. Treatment is started after sending the sputum for culture. The patient comes back 4 weeks later for a check up, and he mentions a pins and needles sensation in his hands. What drug is causing this type of side effect?

Isoniazid

A 60-year-old man presents with a 1-month history of weakness, an inability to climb stairs, and erectile dysfunction. He is not on any medications and has no prior admissions. He has smoked 2 packs of cigarettes a day for the past 45 years. On examination, the strength in his shoulder and pelvic girdle muscles is 3/5. The deep tendon reflexes are decreased. What is the most likely diagnosis?

Lambert-Eaton myasthenic syndrome

An 8-month-old infant presents with shortness of breath, wheezing, intercostal retractions, and respiratory rate of 50. What would further suggest a diagnosis of bronchiolitis?

Liver and spleen palpable

A 45-year-old man presents with a 2-year history of worsening dyspnea and a 1-month history of dry cough. The patient gives no history of fever, chills, chest pain, or wheezing. History is significant for smoking (25 cigarettes / day for more than 22 years). A chest X-ray shows hyperinflated lungs with bullae, tubular heart, flattened diaphragm, and no areas of consolidation. Pulmonary function tests (PFT) reveal a decrease in forced expiratory volume in 1 second (FEV1) along with reduction of FEV1/FVC (forced vital capacity) ratio. These findings are characteristic of what condition?

Emphysema

A 3-year-old boy has been on antibiotics for 3 days for pneumonia. In the past 24 hours, he has developed dyspnea, fever, chills, and left-sided chest pain. On exam, he has diminished breath sounds and dullness to percussion on the left. A chest X-ray shows fluid in the pleural space. A decubitus X-ray does not layer out. What is the most likely diagnosis?

Empyema

You have just finished your evaluation of a 67-year-old man who presented with dyspnea and chest pain 3 days after flying home from Europe. Work-up reveals a right lower pulmonary embolism. Your patient has required 6L of O2 to maintain a saturation above 90% and has continued to remain tachycardic. The remainder of his past medical history is unremarkable. What therapy should you initiate at this time?

Enoxaparin Sodium (Lovenox) plus Warfarin (Coumadin)

A 3-year-old child presents with sudden onset of high grade fever, sore throat, pain during swallowing, and drooling of saliva. There is no history of cough. The child appears toxic and is dyspneic with inspiratory stridor. The child is sitting upright, leaning forward with chin up and mouth open. Suprasternal and intercostal retractions are present. Chest is clinically clear. Blood count shows polymorphonuclear leukocytosis. Lateral radiograph of upper airway shows 'thumb sign'. What is the most likely diagnosis?

Epiglottitis

A 23-year-old woman presents with an anaphylactic reaction after being stung by several bees. She complains of wheezing and shortness of breath. On examination, the client is in acute distress. BP is 98/56 mmHg, PR 110/min, RR 28/min, and temperature 98.7°F. She is immediately treated with supplemental oxygen. In treating this condition further, what drug is required most urgently?

Epinephrine

A 28-year-old woman of Norwegian descent presents with cough, dyspnea, joint pain, fever, fatigue, and weakness. Her history is negative for occupational and environmental exposure. A previous tuberculin skin test was negative. Chest X-ray shows interstitial infiltrate and bilateral hilar lymphadenopathy. Ophthalmological investigation (slit lamp) reveals a clinically silent uveitis. Blood tests show hypercalcemia, and transbronchial biopsy shows non-caseating granulomas. Therapy with systemic corticosteroids is initiated. What skin lesions would be most likely revealed in this patient's legs upon physical examination?

Erythema nodosum

A 16-year-old boy presents with a dry cough of 1 week duration, pharyngitis, and malaise. Examination shows a low-grade fever (38.5° C), and chest auscultation reveals sibilant rhonchi. Chest films show a patchy bronchopneumonia. What is the best course of treatment for this clinical picture?

Erythromycin

On Thanksgiving day, a 5-month-old child presents with wheezing, rapid respirations (>45 breaths/min), and chest retractions. During the last 2 days, the patient had rhinorrhea and a low-grade fever. Breath sounds are normal and there is no cyanosis. Based on the most likely diagnosis, what is the treatment of choice?

General supporting measures

A 16-year-old girl presents with shortness of breath, wheezing in her chest, and swelling of the lips that started several minutes after she had a snack. She had similar symptoms several months ago at a Chinese buffet. She appears restless. Her face is erythematous, and her lips and tongue are swollen. Her blood pressure is 90/60 mmHg, and heart rate is 110/min, rhythm regular, with respirations 26/min and shallow. On auscultation, diffuse wheezing is heard. What is your next step?

Give Epinephrine

A 24-year-old man undergoes a routine medical check-up to become a volunteer in the ER. PPD skin test shows a diameter of 9 mm. Sputum and chest X-ray were negative for tuberculosis. He is otherwise healthy and shows no systemic effect of Mycobacterium tuberculosis infection. What is the most appropriate explanation for the doubtful tuberculin test in this patient?

He was given BCG vaccination earlier in life

A 55-year-old woman presents with a several-month history of increasing cough and dyspnea. She also has increased serum urea, nitrogen, and serum creatinine. A chest X-ray shows multiple bilateral small nodules. A renal biopsy shows a focal necrotizing vasculitis; her antineutrophil cytoplasmic autoantibody (ANCA) test is positive at 1:160. What additional finding would be most likely to occur?

Hemoptysis

A 40-year-old Asian-American man presents with a 3-day history of nausea and vomiting. He notes his health is good. He was started on a new medication 1 month ago for a positive PPD. What is a true statement regarding this patient?

His AST and ALT would be elevated

A 56-year-old man presents for a routine follow-up regarding his positive HIV status. He is compliant with his medications and has been feeling well. In addition to his antivirals, he takes daily trimethoprim/sulfamethoxazole for Pneumocystis jiroveci pneumonia (PCP) prophylaxis. What detail of his history would warrant the prophylaxis?

History of previous PCP infection

A 3-year-old boy presents with a loud cough. Two days ago, the boy developed a runny nose and irritability; the next day, he began to cough loudly and felt warm. His mother tells you that the cough sounded like a wounded animal or a dog barking. The child is diagnosed with croup, and humidification is prescribed. What organism is most likely responsible for the patient's croup?

Parainfluenza virus

A 1-year-old boy presents with 'barking' cough and inspiratory stridor. What virus is the probable causative agent of the infant's illness?

Parainfluenzavirus

A 26-year-old man who has AIDS presents with rapidly progressing dyspnea. The symptoms became apparent following a diagnostic thoracentesis for a left-sided pleural effusion; you suspect the effusion is caused by mycobacterium. The patient reports left-side chest pain and difficulty in breathing. On examination, he is in respiratory distress; temperature is 38°C, RR is 39/min, and BP is 70/40 mm Hg. He has decreased breath sounds in his left lung field, and the percussion note is more resonant. What is the most appropriate course of action?

Perform needle decompression

A 6-year-old boy presents with fever and cough. He has history of several episodes of pneumonia. A sweat test reveals an increased amount of chloride, indicating that he has cystic fibrosis. He is coughing up thick, greenish sputum. Temperature is 37.6° C. A Gram stain of the sputum reveals Gram-negative rods and a culture grows a Gram-negative rod that is oxidase-positive and produces a blue-green pigment. What is the most likely cause of the infection?

Pseudomonas aeruginosa

A 63-year-old woman is 4 days status post partial colectomy for colon cancer. Until now, her postoperative course has been uncomplicated, but today she is experiencing new onset of shortness of breath and chest pain. Her pulse is 120; respirations are 20, temperature is 38.6, and BP is 98/72. On examination, heart rate is regular, but rapid. Her lung sounds are diminished bilaterally; some wheezing is heard. Her abdomen is soft and mildly tender, with a well-healing midline incision. EKG reveals sinus tachycardia. What is the gold standard test for her suspected condition?

Pulmonary angiography

A 33-year-old African-American woman with no significant past medical history, who is in her 38th week of a normal pregnancy, presents with a 1-hour history of shortness of breath. She does not recall any precipitating activities or events that may have provoked these symptoms and recalls that she was laying in her bed for several hours since waking when the symptoms developed. She admits to an associated sharp, nonradiating pleuritic chest pain, as well as lower extremity swelling, which she states has been "persistent throughout the course of her pregnancy". She denies palpitations, chest pressure, cough, sputum, fever, chills, changes in weight, rashes, diaphoresis, abdominal pain, nausea, or a history of allergies. Her physical exam is noteworthy for tachypnea and tachycardia; however, the rest of the vital signs are normal. Her lungs are clear to auscultation bilaterally, without wheezing, rhonchi, or crackles. Her lower extremities are remarkable for 2+ pitting edema up to the level of her knees; there is no calf tenderness, venous cords, or Homan's sign appreciated. Her skin and mucous membranes were without diaphoresis or cyanosis. A bedside EKG performed reveals sinus tachycardia at 120 bpm with prominent S waves in lead I and Q waves in lead III. What is the most likely diagnosis?

Pulmonary embolism

A 45-year-old man presents with a 30-minute history of chest pain; it began while he was on a long airline flight. He denies any trauma to the chest. On exam, his pulse is 110 BPM; respirations are 40/min. The chest radiograph reveals a wedge-shaped opacity. What is the most likely cause of his chest pain?

Pulmonary embolism

A patient recovering from hip surgery begins to ambulate for the first time about 2 hours postoperatively. Suddenly, she experiences shortness of breath. She becomes tachypneic and tachycardic; she experiences pain on inspiration. Prior to getting out of bed, her postoperative course was unremarkable. There is no swelling; there is no palpable thrill at the incision site. What is your initial diagnosis?

Pulmonary embolism

A 44-year-old healthy man, who is a non-smoker, has a 3-cm coin lesion in the right upper lobe. The lesion was revealed on his chest radiograph. The patient did not have any clinical symptoms. The fine-needle aspiration did not reveal any cells. What is the most likely tumor in this case?

Pulmonary hamartoma

A 35-year-old woman presents with a progressive 5-hour history of shortness of breath, cough, and wheezing. This morning she felt that she was 'catching a cold' because of fever, sore throat, and thin, purulent rhinorrhea for which she took aspirin. Her past medical history is significant for persistent rhinitis resistant to therapy; this was diagnosed as nonallergic rhinitis with eosinophilia. What finding on the examination will confirm your diagnosis?

Nasal polyposis

A 67-year-old man presents with a 3-week history of increasing shortness of breath; it occurs even while he is at rest. The patient was diagnosed with congestive heart failure in the past year, and he has been well controlled on oral medication. He has no history of tobacco use. He has gained 10 pounds since his last exam 2 months prior to presentation. On physical exam, there are diminished breath sounds and decreased tactile fremitus bilaterally at the base of the lungs. Dullness to percussion is also noted in the same area. He has 3+ bilateral pitting lower extremity edema. Based on the patient's physical exam and history, what is the most likely diagnosis?

Pleural effusion

A 63-year-old man with a past medical history of hyperlipidemia presents with an 8-month history of intermittent flushing and warmth sensation to his face and neck areas, alternating watery, frothy, and bulky stools associated with steatorrhea, weight loss, and wheezing. His urine demonstrated increased 5-hydroxyindoleacetic acid (HIAA) levels. He had subsequently been referred for an octreotide scintigraphy, which revealed innumerable foci of intense activity scattered throughout the liver. What agent is highly effective at reducing symptoms?

Sandostatin (Octreotide)

A 59-year-old man presents with a 3-month history of progressive exertional dyspnea. He also complains of dry cough, but he denies any history of fever, chest pain, or weight loss. You discover that he works in the ceramic industry at the outskirts of town. He is a non-smoker and drinks alcohol very occasionally. Examination reveals bibasilar crackles, and pulmonary function test indicates FEV1 of 67%, FVC of 73%, and TLC of 75% with DLCO of 65%. Chest X-ray shows "eggshell calcification" of hilar lymph nodes. What disease is this patient at the highest risk of developing?

Silicosis

A 59-year-old man presents with a 3-month history of progressive exertional dyspnea. He has been experiencing dry cough, but denies any history of fever, chest pain, or weight loss. On further questioning, you discover that he works in the ceramic industry at the outskirts of town. He is a non-smoker and drinks alcohol very occasionally. Examination reveals bibasilar crackles, and pulmonary function test indicates FEV1 of 67%, FVC of 73%, and TLC of 75% with DLCO of 65%. Chest X-ray shows "eggshell calcification" of hilar lymph nodes. What is the most likely diagnosis?

Silicosis

A 43-year-old man without any significant PMHx presents with acute-onset of a productive cough, shortness of breath, pleuritic chest pain, and fever. His sputum is described as "thick, brown-colored, and mucoid," but is without blood. He also notes associated fatigue and night sweats. He denies chills, changes in weight, a history of travel, sick or confined contacts, exposure to animals, cigarette smoking, otalgia, sore throat, swollen glands, abdominal pain, diarrhea, rashes, myalgias, and arthralgias. His physical exam is remarkable for fever, tachycardia, and tachypnea. The lung exam is noteworthy for right lower lung field increased tactile fremitus, dullness to percussion, inspiratory crackles, and bronchial breath sounds. What is the most likely etiologic agent responsible for this patient's presentation?

Streptococcus pneumoniae

A 66-year-old man with moderately well-controlled type II diabetes mellitus presents for the management of community-acquired pneumonia. He is currently being treated with intravenous levofloxacin. He is afebrile; pulse is 93, blood pressure is 130/90 mm Hg, respirations are 18/min, and oxygen saturation is 92% on room air. What is the most common cause of bacterial pneumonia among non-institutionalized individuals with no significant co-morbid conditions?

Streptococcus pneumoniae

A woman is diagnosed with primary tuberculosis. She is 5 months pregnant. What drug should be avoided in this patient?

Streptomycin

A 38-year-old man presents with pneumonia. He is a construction worker who is otherwise healthy and has no allergies. His temperature is elevated, he has a nonproductive cough, and he states he "feels bad" and lacks energy. What is the best course of action?

You obtain a specimen for a Gram-stain and sputum cultures and prescribe erythromycin 500mg PO q6h for 14 to 21 days for an empiric therapy against Mycoplasma pneumoniae

A 55-year-old woman presents with constant fatigue. She has experienced multiple episodes of falling asleep at work and dozing off while waiting at red lights. Her husband consistently complains about her snoring and snorting while asleep; it wakes him at night. On physical exam, she is 5'2" and 205 lbs. Her blood pressure is 150/90 mm Hg, her pulse is 82 BPM, her respirations are 16/min, and she is afebrile. The rest of her physical exam is unremarkable. Laboratory studies, including thyroid function tests, are within normal limits. After advising her on diet and weight loss, you schedule her for an overnight sleep study. What results do you expect the study to show?

5 or more episodes of apnea and/or hypopnea per hour

A 31-year-old HIV-positive woman presents for ongoing care. She was diagnosed with HIV 2 years ago, and she began antiretroviral therapy. Her CD4 T cell count is 400 cells/mL, and she has a history of oral candidiasis. As part of her evaluation, a tuberculin skin test (TST) is performed using 5 TU of purified protein derivative (PPD). The test site is examined 48 hours later and the skin reaction is measured. What is the minimum diameter of induration at which this test result should be considered positive in this patient?

5mm

A 6-year-old boy with Down's syndrome is brought to the emergency department after a drowning accident in a lake. Submersion time is unknown, although he was missing for some time. He was resuscitated by his rescuers and intubated by emergency medical technicians during transport. On arrival, his heart rate is 76 beats/min, BP 104/72 mm Hg, and rectal temperature is 82 F (28° C). He remains comatose, exhibiting non-purposeful flexion withdrawal to pain, His pupils are equal and reactive. What is the best way of treating this drowning-related hypothermia?

Active internal warming

A 33-year-old man presents with shortness of breath, wheezing, mild fever, and fatigue. He has had several similar episodes in the past, and each previous episode began after a cold that moved into his chest. Over the past several weeks, he has had a productive cough most mornings. He smokes on a social basis. What is the most likely diagnosis?

Acute asthmatic bronchitis

A 28-year-old man with a history of crack cocaine abuse was rushed in to the emergency room. His mother found him in his apartment. He was cyanotic and severely short of breath. No other history is available. His examination revealed a young man in severe respiratory distress with temperature 99.2° F, pulse 102/min, respiration 40/min, BP165/95 mm Hg, and a pulse oximetry of 66%. He was intubated at the site by the EMS personnel because of lack of improvement of pulse oximetry on 100% non-rebreathing mask. His chest X-ray shows bilateral alveolar and interstitial infiltrates. The arterial blood gas on mechanical ventilation (set at a rate of 12 cycles/min, tidal volume 500 mL) is as follows: PAO2/Fi02 ratio = 100 mmHg, pH 7.52, PCO2 30, PO2 55, and O2 saturation 88%. What is the most likely diagnosis?

Acute respiratory distress syndrome

A 28-year-old man with a history of crack cocaine abuse is rushed in to the emergency room. The mother found him in his apartment; he was cyanotic and severely short of breath. No other history is available. His examination reveals a young man in severe respiratory distress with temperature 99.2° F, pulse 102/min, respiration 40/min, BP165/95 mm Hg, and a pulse oximetry of 66%. He was intubated at his apartment by EMS personnel because of lack of improvement of his pulse oximetry on 100% non-rebreathing mask. His chest X-ray shows bilateral alveolar and interstitial infiltrates. The arterial blood gas on the mechanical ventilation (set at a rate of 12 cycles/min, tidal volume 500 mL) shows the following: PAO2/Fi02 ratio = 100 mmHg, pH 7.52, PCO2 30, PO2 55, and O2 saturation 88%. What can be done to improve his oxygenation at this time?

Add positive end expiratory pressure

A 60-year-old woman presents with a history of persistent cough. She is confined to her bed; walking over 10 paces causes severe breathlessness. She has no energy to carry out any of her regular activities. She has never smoked, and she drinks the occasional glass of wine. On physical examination, she is found to have decreased breath sounds and dullness to percussion over her right lower thorax. Further evaluation reveals an irregular mass in the periphery of the right lung base with a right sided pleural effusion. A needle is inserted into the pleural space and divulges blood stained fluid. If results prove to be a malignancy, what is the most likely sub-type considering she has never smoked?

Adenocarcinoma

A 3-year-old girl presents with a 2-day history of a sore throat and fever. This morning, she was hoarse and seemed to be having more difficulty breathing. On exam, she appears to be in distress and has an oral temperature of 100.0°F. Tympanic membranes are pink but not bulging. Nares are patent without rhinorrhea. She has a barking cough, stridor at rest, and nasal flaring. What treatment is most appropriate in the care of this child?

Admit patient, start humidified oxygen and intermittent racemic epinephrine

A 25-year-old man presents at the hospital after a car accident. He is intubated and placed on a ventilator. He becomes progressively difficult to oxygenate despite increasing the PEEP and the oxygen supply to 100%. Patient remains afebrile. He dies several days later. At autopsy, the lung shows diffuse hyaline membranes in the alveoli, thickened alveolar walls, and many alveolar macrophages, but few neutrophils. What condition did this patient have?

Adult respiratory distress syndrome

A 60-year-old man with history of heavy smoking and moderately severe chronic obstructive pulmonary disease (COPD) has been feeling weak recently. He notes a 3- to 4-day history of cough, chills, pleuritic chest pain, and low-grade fever. Chest X-ray shows a small dense infiltrate in the right lower lobe. Gram stain of the patient's sputum reveals numerous Gram-negative cocci, many of which occur in pairs. What is the most appropriate treatment in this case?

Amoxicillin and clavulanic acid

A 58-year-old man presents to the emergency department for a 4-hour history of chest pain and shortness of breath. He is a long-haul truck driver and noted the symptoms started while he was driving. He admits a mild cough, with some blood in his sputum. The chest pain seems to be associated with breathing and gets worse with deeper inspiration. He denies fever or chills. He is a smoker. He reports no known medical conditions, no medication use, and no prior surgeries. On physical exam, the patient is mildly obese, tachypneic (respiratory rate of 22), and tachycardic (pulse of 112). He appears to be in mild distress. Lungs are normal to auscultation and percussion. Heart exam is normal. His left lower leg has some dependent edema and tenderness. The patient thinks he strained a muscle or bumped it, but he had been too worried about his shortness of breath and failed to mention in his history. The remainder of his exam is normal. He was immediately placed on oxygen at arrival. Several tests results are available. Oxygen saturation: 92% on oxygen Complete blood count: Normal Comprehensive metabolic panel: Normal Electrocardiogram (ECG): Normal Troponin and CK-MB: Normal Chest x-ray: Normal D-dimer: Elevated Assuming all modalities are available to this patient, which of the following interventions is most appropriate for his current condition?

Anticoagulation

During a routine X-ray examination for employment insurance purposes, the radiologist notices a lesion on the right upper lobe of the pulmonary X-ray of a middle-aged man. The patient was treated for pulmonary cavitary tuberculosis (TB) 2 years ago; he has completed treatment, and he has not had any problems since. Refer to the image (solid Easter egg mass in upper right lobe). What late complication of TB is seen in this patient?

Aspergilloma

A 35-year-old woman presents with 5-hour history of progressive shortness of breath, cough, and wheezing. This morning she felt that she was "catching a cold" because of sore throat and thin purulent rhinorrhea, for which she took aspirin. Her past medical history is significant for persistent rhinitis resistant to therapy. What should your patient do to prevent future asthma attacks?

Avoid aspirin

A 23-year-old man presents with a 3-week history of fever (maximum of 103°F), cough, and shortness of breath. He was prescribed trimethoprim-sulfamethoxazole, but it has not alleviated his symptoms and now he is experiencing chest pain due to coughing so much. He remembers that he had a sore throat 4 weeks ago, just prior to the beginning of the symptoms. He is married and denies any extramarital affairs. He has not traveled abroad recently and denies using any illicit drugs. He has no known drug allergies. Given the likely diagnosis, what antibiotic is the most appropriate first-line therapy?

Azithromycin

A 73-year-old woman presents to you for the first time for an initial history and physical. She states that over the past year she has been hospitalized twice for pneumonia that required mechanical ventilation for 2-3 weeks each episode and bacteremia requiring several weeks of antibiotics. Which of the following nail abnormalities would you most likely expect to see?

Beau's lines

A 53-year-old man with a 40-pack/year smoking history presents with a 10-month history of an intermittent cough with productive sputum. He admits to progressive exertional shortness of breath, which recently has limited his activity to climbing 1 flight of stairs or walking 3 city blocks. He denies diaphoresis, fever, chills, chest pain, palpitations, audible wheezing, pleurisy, peripheral edema, hemoptysis, abdominal pain, reflux, regurgitation, diarrhea, melena, or hematochezia. He also denies travel, sick contacts, and drug or alcohol use. His general survey reveals an overweight male with an odor of smoke and nicotine staining of his fingernails. His nails also demonstrate digital clubbing. His pulmonary exam reveals prolonged expiratory phase expiratory, barrel chest, poor diaphragmatic excursion, and wheezing to auscultation. Pulmonary function testing shows airflow obstruction with a reduction in FEV1 and FEV1/FVC ratio; an increase in total lung capacity, functional residual capacity, and residual volume were noted. What intervention has been demonstrated to influence the natural history of this patient's illness?

Behavioral modification - smoking cessation

An 87-year-old woman presents with progressive shortness of breath. She has been in a wheelchair for 15 years due to paralysis of her lower extremities from unknown causes. At this time, she is unable to transfer from the chair to her wheelchair without having dyspnea. She is extremely tired, but denies chest pain, palpitations, cough, hemoptysis, dysphagia, hoarseness, or sick exposures. She has never smoked. Her past medical history is positive for hypertension (treated with enalapril), heart failure, chronic kidney disease, hepatitis C, breast cancer, s/p lumpectomy, and radiation treatment 10 years ago. You order a chest X-ray. What in her medical history would lead you to suspect an EXudative pleural effusion?

Breast cancer. Other causes likely to cause TRANSudutative (chronic kidney disease, heart failure, or cirrhosis)

A woman is leaving with her 1-year-old son to go grocery shopping. She lives in a condominium with a small covered garage. As she starts her car, her telephone rings. She runs into the house to answer the phone, leaving her son safely buckled into his car seat and the car engine still running. When she returns to her car, she finds her son still buckled into his car seat. He is unconscious. She calls the paramedics. They tell her that the boy was exposed to noxious fumes. What will the child's mother notice about his skin?

Cherry-red (hyperemic)

A 25-year-old man presents with acute onset shortness of breath associated with right-sided chest pain. The pain is unaffected by position and is worse with inspiration. He was grocery shopping when it started. He denies chest trauma. He had an upper respiratory infection earlier in the month that had resolved without incident. He smokes 1 pack of cigarettes per day, and he has no significant past medical history. On examination, he is afebrile and BP is138/80 mm Hg; pulse is 124, respiratory rate is 24, and pulse oximetry is 94% on room air with mild respiratory distress. Trachea is midline. He has increased resonance to percussion with no breath sounds on the right anterior apex; the other lung fields are clear to auscultation. Heart is tachycardic with normal S1 and S2; no murmur, rubs, or gallops are present. What is the imaging of choice to make the diagnosis?

Chest Radiograph

A 60-year-old man presents with a 2-year history of shortness of breath. He has been a heavy smoker for 40 years. He has a chronic cough; it produces at least ½ a cup of mucoid sputum a day. He was hospitalized 3 times last year for infective respiratory disease. He is currently on asthmatic medication; he is taking 5mg salbutamol, 250-mcg ipratropium 4 hourly by nebulizer, slow release theophylline 300-mg b.i.d., and prednisolone 10 mg daily. At present, he presents with history of worsening of cough and shortness of breath. On examination, the patient is cyanotic, T 100, P 100/m, and RR 25. What should the initial investigation be?

Chest X-ray

A 30-year-old woman presents because she recently had a PPD skin test; the transverse diameter of the induration was 14mm. The patient denies ever having tuberculosis and she is asymptomatic now. However, she expresses some anxiety about the result of the skin test because, for the last 6 months, she has worked as a nurse for a home resident. Her patient is a vent-dependent tetraplegic. She remembers having the BCG vaccine in childhood. On clinical examination, there are no abnormalities. What is the most appropriate next step?

Chest x-ray

A 4-year-old boy presents with a 1-month history of weight loss, fevers, cough, and night sweats. He and his family moved to the United States from Africa 3 months ago. He is a thin, pale boy in no acute distress. His heart rate and rhythm are regular; his lungs are clear to auscultation, and he has no organomegaly. What initial tests would most likely have the most value?

Chest x-ray and tuberculin skin test

A 25-year-old man presents with fever, cough, and shortness of breath; he is experiencing chest pain. He had a sore throat 2 weeks ago. He is married, denies any extramarital affairs or use of illicit drugs; he has not traveled abroad in the recent past. You suspect pneumonia. What is the most likely causative agent?

Chlamydophila pneumoniae

A 53-year-old woman presents with a 2-year history of chronic cough. The cough produces large volumes of grossly purulent sputum. She has a history of recurrent respiratory infections; they resulted in 5 hospitalizations in the past year. She also had similar complaints during the previous year. Shortness of breath limits her daily activity considerably. Upon pulmonary examination, bilateral breath sounds are audible, with inspiratory and expiratory crackles at the lung bases. Chest X-rays reveal increased lung volumes, non-flattened diaphragm, and thickened bronchial walls. What is the initial diagnosis?

Chronic bronchitis

A 55-year-old woman presents with dyspnea, cough, and a feeling of fullness in her head. Clinical examination shows facial and neck swelling, dilated venous channels over the upper part of her body, facial flushing, and cyanosis. She was recently diagnosed with adenocarcinoma of the lung. Her symptoms are most probably of what nature?

Compressive

An 84-year-old woman, with a past medical history of myocardial infarction, congestive heart failure, dyslipidemia, asthma, and lung cancer, presents with a 6-hour history of dyspnea, nonproductive cough, hemoptysis, and a "sharp, stabbing" pleuritic chest pain. She has a 50-pack/year smoking history, but she quit 10 years ago; she denies any alcohol or illicit drug use, sick contacts, or recent travel. She also denies any recent hospitalizations or surgeries. Her review of systems is negative for any fever, chills, palpitations, wheezing, abdominal pain, nausea, vomiting, diarrhea, and rashes. Her physical exam is notable for abdominal distension, hepatosplenomegaly, supraclavicular lymphadenopathy, and right greater than left lower extremity pitting edema. The cardiac exam reveals tachycardia, jugular venous distension of six cm, and an S3 gallop; the lung exam is notable for tachypnea, diffuse dullness to percussion, decreased tactile fremitus, and absence of breath sounds. The skin exam does not reveal any abnormalities. Pleural fluid determination from diagnostic thoracentesis notes a clear pleural fluid with an LDH level of 160 units/liter, a pleural-to-serum protein ratio of 0.2, and a pleural-to-serum LDH ratio of 0.3. Pleural fluid glucose, leukocyte, and pH assessments are found to be within the normal range; no red blood cells are identified. What is the most likely contributory etiology based upon this patient's presentation and diagnostic test result?

Congestive heart failure

A 72-year-old man presents due to worsening shortness of breath, orthopnea, and chest pain; symptoms have been occurring for the past few weeks. The patient admits to some chronic heart problems as well as fatigue, dyspnea, and a non-productive cough; however, he feels like symptoms have worsened recently. He denies fever, chills, and a productive cough. On physical exam, the man has mildly increased respiratory effort, but he does not appear in distress. He is barrel-chested. His breath sounds are diminished bilaterally, with dullness to percussion over right and left lower lungs. No pleural friction rub is noted. On cardiovascular exam, an S3 gallop and mild tachycardia (110 bpm) are noted. Clubbing of the fingers, dependent edema in the lower extremities, and jugular venous distention are also noted. His cardiac enzymes and electrocardiogram demonstrate no acute cardiac pathology. Pleural fluid and cardiomegaly are found on the chest X-ray, and a thoracentesis is performed. The pleural fluid is generally clear in color, testing negative for chylomicrons and triglycerides. It has low levels of red blood cells, white blood cells, protein, and lactate dehydrogenase (LDH). What should be recommended as most useful for this patient as a tertiary level of preventive medicine?

Control of his heart failure to prevent pulmonary complications

A 62-year-old man presents with a 2-month history of worsening fatigue and shortness of breath. He has a past medical history of emphysema; it is attributable to his 85-pack/year cigarette smoking history. The patient complains of nearly passing out while climbing the stairs in his house. He tells you that he feels like his heart races. He reports chronic shortness of breath and cough, but he now he feels like his dyspnea is dramatically worse; he can no longer sleep in his bed. He has been trying to sleep propped up in a chair at night. He is also experiencing fatigue. He has gained about 15 pounds, and he notes that he can no longer lace up his shoes. He denies fever, chills, and chest pain. His cough produces some mucus, but no hemoptysis. His vitals are shown in the table. Weight: 212 lb Height: 69" Body mass index: 31.3 Pulse: 108 Blood pressure: 140/88 Temperature: 98.2°F Pulse oximetry: 88% On physical exam, you see a man in mild respiratory distress; he is sitting upright and leaning forward, and he uses accessory respiratory muscles for breathing. The exam is significant for reduced air movement and mild rales bilaterally in the lungs; distended neck veins; mild tachycardia with prominent P2; lower extremity edema; and right upper quadrant abdominal tenderness with hepatomegaly. Based upon this patient's history and physical, what is the most likely diagnosis?

Cor pulmonale

A 36-year-old African-American woman presents with a nonproductive cough, malaise, mild fever, and mild dyspnea. A chest X-ray demonstrates a right hilar mass. A pulmonologist is consulted and performs a biopsy during bronchoscopy. The report reveals that the mass is a non-caseating granuloma. What therapeutic should be prescribed?

Corticosteroids

A 3-year-old boy presents with difficulty in breathing and a cough that sounds like a seal. On examination, the child has fever, a harsh barking cough, a respiratory rate of 38/minute, and minimal stridor on agitation. On lung auscultation, there are no rales or wheezing. On cardiac auscultation, there is tachycardia. Radiological examination reveals the so-called 'steeple sign'. What is the most likely diagnosis?

Croup

A 10-year-old girl presents with recurrent bronchitis. Her past history is significant for polypectomy, nasal polyps, failure to thrive, and repeated attacks of bronchitis. Examination reveals clubbing and diffuse rhonchi on respiratory auscultation. Investigations reveal subnormal lung function, decreased exercise tolerance, and a sweat chloride concentration of 62mEq/l (Normal <40 mEq/l). What is the most likely diagnosis?

Cystic fibrosis

A 2-year-old boy presents with chronic respiratory infections, foul-smelling pale stools, weight loss, and wheezing. His mother reports using moist mist treatments repeatedly, and they offer some relief. A clinical exam shows delayed growth, an enlarged spleen upon palpation, and clubbing of fingers and toes. Auscultation of the lungs demonstrates pulmonary congestion. Lab results report a positive fecal fat test and sweat chloride test. What is the most likely diagnosis?

Cystic fibrosis

A 4-year-old boy presents with a recurrent cough. The boy is small for his age, but his mother insists that he eats well. She also mentions that he frequently passes stool that is oily and foul smelling. He has had pneumonia several times, and the doctor notes that he is wheezing. What disorder does the boy have?

Cystic fibrosis

A 14-year-old boy presents with worsening shortness of breath; it most often occurs when he plays soccer. He often awakens in the middle of the night due to 'attacks': he starts to feel anxious because he feels like he cannot breathe, and he experiences chest tightness. He suffers from a dry cough, especially after playing sports. His mother has put a humidifier in his room and has him use his sister's inhaler, which seems to help temporarily; the boy has been using it 5 - 6 times daily. The family history is significant for asthma in his sister, father, and 3 other paternal relatives. The patient and his mother are not aware of any allergies. He denies fever, chills, and chest pain. In between 'attacks', he feels well and normal. The patient's past medical history is noncontributory. There are no known medical conditions; he has no drug allergies, and he has not had any surgeries. Other than the aforementioned inhaler, he does not take any medications. In addition to his own albuterol inhaler, what medication should be prescribed for this patient?

Daily low-dose inhaled budesonide

A 76-year-old Caucasian man with a 90 pack per year smoking history presents with progressive fatigue, tachypnea, exertional dyspnea, cough, and lower extremity edema. Inspection of his chest and abdomen reveals an increased chest diameter, labored respiratory efforts with retractions and cyanosis, left parasternal and subxiphoid heaves, hepatojugular reflux, and a pulsatile liver. Additionally, there is scattered wheezes and crackles in his lungs and bilateral lower extremity edema. What physical exam findings would be most consistent with the underlying diagnosis?

Distended neck veins with prominent a or v waves

A 40-year-old man with no significant past medical history presents with a 2-day history of alternating fever and rigors, diaphoresis, fatigue, and a productive cough. He admits to mucoid sputum of moderate quantities. He denies a history of smoking, alcohol use, recent travel, or sick contacts. He further denies chest pain, palpitations, hemoptysis, rashes, abdominal pain, nausea, vomiting, or diarrhea. On physical exam, he is found to be tachypnic and was observed to be intermittently coughing. The pulmonary exam was notable for bronchial breath sounds over the right anterior 4th, 5th, and 6th intercostals spaces. A chest radiograph revealed a right middle lobe consolidation. What additional physical exam findings would be consistent with this patient's most likely diagnosis?

Dullness to percussion

A 64-year-old man with hypertension, coronary artery disease, and poorly-controlled left ventricular congestive heart failure presents with a 3-day history of insidious chest pain. Pain is made worse when he takes a deep breath in and when he coughs. He denies any relation of pain to position, activity, or food intake. He denies fever, chills, palpitations, sputum production, wheezing, abdominal pain, nausea, vomiting, diarrhea, or peripheral edema. His physical exam reveals a widespread friction rub upon inspiration, absent lung fremitus, and reduced lung sounds over the thoracic cavity. What additional physical exam finding would be most likely expected in this patient?

Dullness to percussion

A 67-year-old Caucasian woman presents with a 5-hour history of intense shortness of breath at rest. She is very anxious because she feels like it is getting more difficult to catch her breath. She admits to getting home yesterday from a 2-day car ride after visiting her grandchildren. Based on the history, physical examination, and diagnostic studies done thus far, there is a moderate to high probability of this patient having an acute pulmonary embolism. A confirmed diagnosis is not yet obtained. When should parenteral anticoagulation therapy be initiated?

During the diagnostic evaluation

During your well newborn morning nursery rounds, a nurse tells you about an abnormal finding on an antenatal ultrasound that was done. The rest of the maternal history is unremarkable. The infant was born last night. She was born at term and without any complications, and she seems to be feeding and transitioning well; however, she has not passed stool yet. What antenatal ultrasound finding might be a marker for cystic fibrosis?

Echogenic bowel

A 30-year-old man presents with a 2-month history of coughing, and a 2-day history of coughing blood. He has been losing weight and sweating at night. On physical examination, the patient appears wasted, tachypneic, has bronchial breath sounds in the right upper lobe, and crepitations in the left upper lobe and right mid-zone. His direct sputum result comes back positive for acid-fast bacilli with Ziehl-Neelsen stain. His sputum is sent for culture and treatment is started. Refer to the image (TB). Retrobulbar neuritis is the predominant adverse effect of what drug?

Ethambutol

An immigrant worker presents with a chronic cough, blood-stained sputum, and night sweats. Physical examination reveals a wasted middle-aged man with bronchial breathings on the right upper lobe. His sputum examination under microscope shows acid-fast staining rods, and his PPD is 15 mm. Based on clinical findings, sputum examination, and PPD result, pulmonary tuberculosis is diagnosed. Treatment is started after sending the sputum for culture. The patient comes back 4 weeks later for a check up, and he mentions not being able to see clearly and not being able to distinguish the color blue from green. What drug is most likely causing this side effect?

Ethambutol

A 46-year-old man with type II diabetes mellitus, hyperlipidemia, homocysteinemia, and metabolic syndrome presents with a 5-month history of excessive daytime sleepiness without refreshing sleep, a depressed mood, impaired memory, and an inability to focus at work as well as while driving. Additionally, he has been told by his wife that he snores rather loudly and is restless while sleeping. He also admits to progressive irritability and impotence. Upon physical exam, he is found to be hypertensive and has elevated BMI with abdominal obesity and an enlarged neck circumference. He was found to be falling asleep at several instances during the exam sequence. What is a known risk factor for this patient's underlying condition?

Evening alcohol use

A 68-year-old African-American woman with a past medical history of obstructive sleep apnea, hypertension, and COPD presents due to chronic, progressive dyspnea, which initially occurred upon exertion, but over the last 8 months has also been noted at rest. She is maintained on home oxygen for COPD. There is associated fatigue, substernal exertional chest pain, and 2 episodes of syncope, which occurred during exertion. She denies other symptoms. Physical exam reveals an oxygen saturation of 90%, a left parasternal lift, narrow splitting of the second heart sound, accentuation of the pulmonary component of the second heart sound, an early systolic ejection click, and an S4 gallop. No murmurs are identifiable. There is also +1 pitting edema noted bilaterally to the lower extremities to the mid-calf level. Bloodwork was remarkable for a hemoglobin level of 19 gm/dL and a hematocrit of 68%. An EKG and chest x-ray were performed (The electrocardiogram demonstrates right axis deviation, right ventricular hypertrophy, and right atrial enlargement. The chest x-ray indicates cardiac enlargement, with prominence of the pulmonary artery, right atrium, and right ventricle.). Which health maintenance statement is correct regarding this patient?

Expected complications include syncope, hypoxia, pedal edema, and hepatic congestion. (Cor pulmonale)

A 68-year-old African-American woman with a past medical history of obstructive sleep apnea, hypertension, and COPD presents with chronic, progressive dyspnea, which occurred upon exertion initially, but is now noted at rest over the last 8 months. She is maintained on home oxygen for COPD. There is associated fatigue, substernal exertional chest pain, and 2 episodes of syncope, which has occurred during exertion. She denies other symptoms. Physical exam reveals an oxygen saturation of 90%, a left parasternal lift, narrow splitting of the second heart sound, accentuation of the pulmonary component of the second heart sound, an early systolic ejection click, and an S4 gallop. No murmurs are identifiable. There is also +1 pitting edema noted bilaterally to the lower extremities to the mid-calf level. An EKG and chest x-ray were performed, which revealed the results shown (The electrocardiogram demonstrates right axis deviation, right ventricular hypertrophy, and right atrial enlargement. The chest x-ray indicates cardiac enlargement, with prominence of the pulmonary artery, right atrium, and right ventricle.). What treatment is considered beneficial in the management of this patient at this time?

Furosemide

A 58-year-old woman with a past medical history of hypertension, hyperlipidemia, breast cancer, hip fractures, and coronary artery disease is being evaluated for acute-onset, severe left-sided pleuritic chest pain over the course of the last 2 hours. The pain is associated with feelings of anxiety, hemoptysis, shortness of breath, and nausea. She "feels warm", but denies chills, palpitations, wheezing, edema, vomiting, abdominal pain, abnormal bowel habits, or dietary intolerances. She admits to a 30 pack/year smoking history, but denies drug or alcohol use. Upon physical exam, she is found to be febrile, hypotensive, tachycardic, tachypnic, diaphoretic, and in acute painful distress. There is perioral cyanosis and a pleural friction rub to the left lung fields; the remainder of the exam is normal. What is the most appropriate therapeutic intervention for this patient at this time?

Heparin

A 50-year-old obese woman undergoes a cholecystectomy and T-tube drainage of the common bile duct. On the 7th postoperative day, she develops sudden epigastric and left-sided chest pain. She is short of breath and is sweating profusely. Her temperature has been between 99°F and 100°F for the past 2 days. The patient's vitals are as follows: pulse 90/min, BP 110/60 mm Hg, and RR 24/min. The ECG reading shows tachycardia, and the ventilation-perfusion lung scan shows a VQ mismatch. Based on the most likely diagnosis, what should be the initial management of this patient?

Heparinization (PE)

A 41-year-old woman presents due to worsening symptoms. She was diagnosed with idiopathic pulmonary hypertension about 2 years prior to presentation; she is on home oxygen therapy. She has longstanding fatigue and dyspnea, but she is now experiencing profound dyspnea with exertion; swelling in her ankles; some discomfort in her right, upper abdomen; and the inability to breathe well when lying down. She has always been thin, but her weight has increased by 10 pounds in the last month. She denies fever and chills. She recently had an electrocardiogram (ECG), but she has not seen a healthcare provider to discuss the results. The ECG report indicates peaked p waves, right axis deviation, and tall R wave in V1. Based on this patient's history and test results, what physical exam findings would be expected?

Hepatojugular reflux

A 50-year-old man presents with a 2-week history of not being able to see well. He is not on any medications. He has been smoking 2 packs of cigarettes a day for the past 30 years. On examination of his right eye, there is ptosis and miosis. A chest radiograph reveals a rounded opacity in the right lung field. What is the most likely diagnosis?

Horner syndrome

A 64-year-old man presents with progressive dyspnea, fatigue, chronic dry cough, and exercise intolerance. His symptoms have been progressing over the past several months to a year. As part of your work up, you obtain pulmonary function testing; it reveals a FEV1/FVC ratio of >0.7, a decreased total lung capacity, and a decreased residual volume. What is the most likely diagnosis?

Idiopathic pulmonary fibrosis

A 72-year-old man is evaluated following admission to the hospital for a 1-year history of progressive dyspnea, nonproductive cough, weight loss, low-grade fevers, fatigue, and myalgias. His past medical history is remarkable for atrial fibrillation (for which he takes amiodarone), hypercholesterolemia, and recurrent urinary tract infections, for which his urologist prescribed nitrofurantoin on a chronic, prophylactic basis. He denies cigarette use, a history of murmurs or coronary artery disease, chills, fatigue, rhinitis, otalgia, chest pain, wheezing, hemoptysis, syncope, abdominal pain, rashes, peripheral edema, diaphoresis, arthralgias, vomiting, and urinary complaints. A bedside echocardiogram and electrocardiograms are unremarkable for abnormalities; a chest X-ray revealed peripheral reticular opacities at the lung bases and a generalized honeycombing pattern. What is the most likely diagnosis?

Idiopathic pulmonary fibrosis

Your patient is a 57-year-old Caucasian man who presents with worsening shortness of breath. While obtaining his history, you uncover that he has noted increasing shortness of breath with minor exertional activity, as well as a persistent yet nonproductive cough. The patient admits to being a former smoker with a 34 pack/year history, admitting to cessation at the age of 50. He denies any known caustic occupational exposures and states he worked in an office his whole life. He admits to an uncle having some kind of breathing issues, although he is unsure of a definite diagnosis. Patient denies weight loss, fever, or significant recent illness. Physical examination is pertinent for significant clubbing of the fingers, inspiratory squeaks auscultated during the pulmonary exam, and a right-sided gallop found during the cardiac exam. What is this patient's most likely diagnosis?

Idiopathic pulmonary fibrosis

A 3-year-old girl presents with a rapid onset of high fever and noisy breathing that developed in the last 8 hours. The patient has missed standard immunizations. The child denies having a sore throat, but she tells you that she cannot eat or drink anything because of pain; when she tells you this, she points with her finger to a spot deep inside her mouth. On examination, the child has a muffled voice, drooling, fever, tachycardia, tachypnea, and she adopts a leaning forward position with the neck extended. Stridor and retractions of the chest wall are noted on inspirations, but the lungs are clear on auscultation. During the examination, the child becomes cyanotic and prostrated. What is the most appropriate next step in patient management?

Immediate intubation

On Thanksgiving day, a 5-month-old infant presents with wheezing, rapid respirations (>45 breaths/min), and chest retractions. The patient has a 2-day history of rhinorrhea and low-grade fever. Breath sounds are normal, and there is no cyanosis. What test can confirm the most likely diagnosis?

Immunofluorescence of nasal secretion

A 22-year-old man presents with a 3-day history of sharp, localized, intermittent back pain on the right side. The patient denies any physical trauma to the area. He states the pain intensifies with deep breathing, sneezing, and coughing. He also reports he is experiencing a concurrent viral respiratory illness, for which he has not sought treatment. He denies any PMH other than typical childhood illnesses. His vitals are: BP 125/76 mm Hg, pulse 85/min, temperature 97.8° F, respiration rate 16/min, height 67 inches, and weight 170 lb. Lung exam reveals occasional coarse rhonchi throughout all lung fields, without wheezes or rales. There is no increase or decrease in tactile fremitus in any lung fields, egophony is not present, and diaphragmatic excursion is equal bilaterally. Localized tenderness in the right back at the level of ribs 7, 8, and 9 is present with deep breaths and coughing, but tenderness is not elicited with palpation of the area. Skin exam reveals no rashes or other abnormal findings. The remainder of the physical examination does not demonstrate any other abnormal findings. Based on the most likely diagnosis for this patient, what treatment regimen would be most appropriate?

Indomethacin 25 mg bid, with a cough suppressant, for 1 week

A 67-year-old man presents for a routine follow-up visit. His past medical history includes chronic obstructive pulmonary disease (COPD) for the past 3 years and hypertension (HTN) for the past 5 years. Daily medications include a tiotropium inhaler and hydrochlorothiazide. He has smoked a pack of cigarettes a day for 50 years, and he has no desire to quit at this time. Physical exam remains unchanged from 3 months ago, and the patient has no complaints. His last pneumococcal and influenza vaccinations were 2 years ago. What should be offered at this visit?

Influenza vaccine

A 57-year-old man, who is well known to your practice, presents with a 6-month history of a daily productive cough. The patient is a nonsmoker, and he has worked in a local coal mine for the past 39 years. He very rarely comes in to see a healthcare provider. When asked if he is up to date with his vaccinations, he does not recall the last ones he received; he also does not recall when these may have been given. Given the most likely diagnosis at this time, what vaccinations would be recommended to help decrease any significant morbidity and mortality in this patient?

Influenzae and pneumococci

A 22-year-old woman presents with shortness of breath. She has a history of intermittent wheezing while exercising. On examination, you find that her pulse rate is increased; there are diffuse wheezes on pulmonary auscultation. Her oxygen saturation is 95%. What would be the most effective therapy in this patient?

Inhaled albuterol

A 32-year-old man presents due to occasional shortness of breath and associated cough, especially when he is working outside. He has associated chest tightness, which resolves within minutes when he sits down and rests. These symptoms occur 1-2 days a month. He is otherwise healthy and does not smoke. Blood pressure is 128/74 mm Hg, and pulse is 76; respiration is 14, and and pulse oximetry is 100% on room air. His FEV1 is 96%. What is the treatment of choice?

Inhaled short acting beta agonist

You are currently on an inpatient hospitalist team in a local pediatric hospital. First thing this morning, your team is called in to evaluate an infant born at 27 weeks gestation 50 minutes ago. Upon initial inspection of the newborn, you observe rapid, labored grunting respirations, flaring nostrils, and retractions that are present above and below the breastbone. Auscultation reveals diminished air movement, and a chest radiograph reveals a ground glass appearance in the lung fields bilaterally. The patient is diagnosed with respiratory distress syndrome (RDS). What pharmacological agent should be initiated as soon as possible to help alleviate the patient's signs and symptoms?

Inhaled surfactant replacement

A 62-year-old man presents with a 2-month history of worsening fatigue and shortness of breath. He has a past medical history of emphysema; it is attributable to his 85-pack/year cigarette smoking history. The patient complains of nearly passing out while climbing the stairs in his house. He tells you that he feels like his heart races. He reports chronic shortness of breath and cough, but now he feels like his dyspnea is dramatically worse; he can no longer sleep in his bed. He has been trying to sleep propped up in a chair at night. He is also experiencing fatigue. He has gained about 15 pounds, and he notes that he can no longer lace up his shoes. He denies fever, chills, and chest pain. His cough produces some mucus, but no hemoptysis. His vitals are shown in the table. Weight: 212lbs Height: 69" Body mass index: 31.3 Pulse: 108 Blood pressure: 140/88 Temperature:98.2 °F Pulse oximetry: 88% On physical exam, you see a man in mild respiratory distress; he is sitting upright and leaning forward, and he uses accessory respiratory muscles for breathing. The exam is significant for reduced air movement and mild rales bilaterally in the lungs; distended neck veins; mild tachycardia with prominent P2; lower extremity edema; and right upper quadrant abdominal tenderness with hepatomegaly. Based upon this patient's history and physical exam findings, what intervention is the most appropriate next step?

Iniate oxygen

A 27-year-old accident victim with a head injury is admitted to the ICU and kept on mechanical ventilatory support. On the 7th day after admission, he is clinically diagnosed with pneumonia. Blood samples and lower respiratory secretions are submitted to the laboratory for culture, and empiric antimicrobial therapy is started. What is the most likely etiologic agent of pneumonia in this patient?

Klebsiella pneumoniae

A 47-year-old man presents with persistent chills following a recent upper respiratory tract infection. The patient reports cough with the production of yellowish sputum and myalgia. The patient is febrile (37.9C), with a HR of 94 bmp and a respiratory rate of 24/min. Knowing that the patient has had a prolonged exposure to the air conditioned space in a large office building, what may be the causative microbial agent in this case?

Legionella pneumophila

A 68-year-old man presents with long-standing dyspnea and a non-productive cough. The patient has had the cough and progressively worsening dyspnea for about 1.5 years. He denies other symptoms, and he is not a smoker. He denies fevers, chills, and night sweats. He denies unusual travel, hobbies, and occupational exposures. The rest of his review of systems is negative. His past medical history is unremarkable. He had a workup through his family practice, and he was then sent to a cardiologist, who ruled out cardiovascular causes of his dyspnea. He was given trials of various antibiotics, inhalers, and steroids; there was no improvement in symptoms, despite good compliance. Several screening blood tests (including blood count, metabolic panel, HIV, and autoimmune markers) are done, revealing no abnormalities. A chest x-ray shows a few reticular opacities. A high-resolution computed tomography (HSCT) scan shows some scattered areas of reticular opacities and honeycombing. Your office runs a pulmonary function test (PFT), which shows some restrictive impairment and reduced perfusion of carbon monoxide. His physical exam is significant for fine inspiratory bibasilar crackles and clubbing in the fingers. An occasional dry cough is noted. What intervention is recommended for this patient's condition?

Lung transplant

A 69-year-old man presents with a 7 - 10 day history of increasing dyspnea and inspiratory chest pain. He has a 2-pack-per-day smoking history and abuses alcohol. He has hypertension, diabetes, coronary artery disease, and chronic kidney disease. He was discharged from the hospital 2-1/2 weeks ago; he was diagnosed with congestive heart failure after presenting with similar symptoms. The patient's breathing appears labored; there are diminished breath sounds on auscultation and dullness to percussion in the lower 1/2 of the lung fields, bilaterally. A chest X-ray reveals bilateral moderate pleural effusions. Thoracentesis reveals pleural fluid with the following characteristics: (1) turbid in appearance; (2) 2500 white blood cells/microliter; (3) glucose equal to serum levels; (4) ratio of pleural fluid protein to serum protein of 0.75; and (5) ratio of pleural fluid LDH to serum LDH of 0.72. Based on the information above, what is the most likely etiology of the pleural effusions?

Malignancy

A 75-year-old man presents with a 4-month history of dyspnea on exertion and a productive cough. He also unintentionally lost 10 pounds in 2 months. His past medical history is significant for coronary artery disease and a myocardial infarction. He has smoked an occasional cigar the last few years. He has been retired for 12 years, but for 30 years he worked odd jobs in the construction industry. He also helped his father in the family's car garage shop. Vital signs are normal. His physical exam is remarkable for decreased breath sounds in left lower lung fields and dullness to percussion. A chest radiograph is ordered, and it shows a left sided pleural effusion. What condition does this patient have?

Malignant mesothelioma

You receive notice that one of your patients, a 2-month-old boy, was found dead in his parents' bed. Sudden Infant Death Syndrome (SIDS) is suspected. He was just seen for his 2-month well baby check and seemed healthy and thriving, although he was recovering from a mild upper respiratory infection. Everything looked consistent with a mild cold on his exam and he was afebrile. You recall that his mother mentioned she had returned to work part-time and found a family care home for him while she worked. You remember counseling about back to sleep care and other risks for SIDS. Although she is a smoker, she had cut back during her pregnancy and said she now only smokes outside. She seemed receptive, but she was also preoccupied with her 2 other small children present and seemed very tired. What major risk factor in almost every epidemiologic study of SIDS may have influenced this case?

Maternal smoking during pregnancy

An 82-year-old man presents with difficulty breathing and chest pain. He admits to poor follow-up with his primary care physician; he has not been seen by his doctor in several years. When last seen, his health issues included hypertension, hyperlipidemia, diabetes mellitus type II, and being grossly overweight. His prescriptions ran out, so he has not been on any treatment for these issues for several years. Chest X-ray done at the bedside reveals significant bilateral effusions that are initially felt to be secondary to congestive heart failure based on various other findings. A pulmonologist is consulted and performs bilateral thoracentesis to help alleviate the patient's symptoms. The right side is significantly more affected than the left, with over 1000 mL drained versus about 625 mL on the left. During initial testing, what finding was most indicative of these thoracentesis results?

Mediastinum shifted to the left

A 65-year-old man presents with fatigue, shortness of breath, chest pain, and weight loss. Physical exam findings include diminished breath sounds on auscultation, and there is dullness to percussion. You order a CXR and it reveals a pleural thickening and unilateral pleural effusion. Based on the CXR, which of the following cancers is highest on the differential diagnosis?

Mesothelioma

A 50-year-old woman presents with right-sided pleural effusion. Thoracentesis shows the presence of exudative serosanguineous pleural fluid and positive cytology. For what condition is this finding most typical?

Metastatic infiltrating ductal carcinoma

A 54-year-old man with emphysema presents with a blood pressure of 157/101 mmHg. Over the next several months, he is prescribed angiotensin-converting enzyme inhibitors, diuretics, and calcium channel blockers, but the patient has to discontinue each agent because of undesirable side effects. He is instead prescribed a beta antagonist. What beta antagonist would be most appropriate for this particular patient?

Metoprolol

An 18-year-old man has been diagnosed with asthma since he was 12 years old. The patient states he has been controlled using his rescue inhaler but for some reason his allergies this year are really acting up. You ask him how many times a week he has been using his inhaler. The patient responds "4 times a week." He also wakes up multiple times a month due to shortness of breath. He wheezes on a regular basis. His FEV1 is above 80% predicted. You determine you need to add additional therapy at this time. Based on the stepwise approach for asthma, what should you prescribe?

Mometasone furoate

A 52-year-old woman, living a non-sedentary lifestyle, presents with a 5-day history of low-grade fever, flu-like syndrome, sore throat, and malaise. She has to 'catch' her breath because of pain on inspiration and when coughing. She has no known past medical or surgical history; she is not on any medication, and she has no pertinent family history. She denies any medication use, including over-the-counter medicines. On physical examination, her vitals are: temperature 100.6° F, pulse 86/min, BP 133/75 mm Hg, and RR 20 cycles/min. She has shallow breathing, resonant percussion notes, fair air entry with vesicular breath sounds, and friction rub. Her blood gas on room air is as follows: pH 7.36 PCO2 44 mm Hg PO2 100 mm Hg HCO3 26 mEq\L O2 saturation 99.8% Her chest X-ray (CXR) is normal and D-dimer assay is also normal. What is the most appropriate management modality for this patient?

NSAIDs (non-steroidal anti-inflammatory drugs)

A 30-year-old woman with no significant past medical history is evaluated for a 3-month history of exertional dyspnea and increased fatigue; it is associated with chest discomfort and lightheadedness. Her physical exam reveals a right ventricular heave, a widely split S2, an accentuated pulmonic component of S2, a pulmonary ejection click, an S3 gallop, and a tricuspid regurgitation murmur. There is jugular vein distention and liver enlargement, but no peripheral edema. Pulmonary auscultation is normal. An EKG is performed; the tracing is attached. An inhaled vasodilator challenge results in a favorable response. What is the most appropriate initial therapy in this case?

Nifedipine

A 17-year-old boy presents with intermittent bouts of shortness of breath, coughing, and chest tightness. The symptoms most often occur during football practice; sometimes, however, they also occur when he is just standing outdoors in cold weather. He denies palpitations, fever, and chills. Past medical history is noncontributory. He is a nonsmoker. Physical exam is unremarkable. Vital signs are as follows: BMI 19 kg/m2, BP 116/70 mmHg, HR 80 bpm, and SpO2 99% on room air. His physician orders pulmonary function testing (PFTs) and a chest X-ray (CXR). Based on the most likely diagnosis, what chest X-ray finding(s) is most likely?

Normal chest X-ray

A 66-year-old man presents with vomiting. He has lung cancer, but he has declined treatment. His father died of lung cancer at the age of 65. On examination, he is confused and dehydrated with a BP of 100/50 mm Hg and PR of 100 beats/minute. Laboratory analysis reveals a calcium level of 16 mg/dL. His renal function tests are normal. What is the most appropriate initial therapy in the management of this patient?

Normal saline

A 3-year-old boy presents for his annual well child check. He and his family moved to the United States from Africa 4 months ago. He is a thin boy; he is in no acute distress. His heart rate and rhythm are regular; his lungs are clear, and he has no hepatosplenomegaly. Due to his recent immigration, he is given a tuberculin skin test. He has no known chronic medical conditions and no known contacts with tuberculosis disease. The test shows an induration of 6 mm. What is the next step in the management of this patient?

Normal well child care

A 12-year-old boy presents with sudden onset of dyspnea with wheezing. The patient had a similar appearance a month ago. An arterial blood gas shows hypoxemia, hypercapnia, and acidosis. The chest X-ray shows clear lung fields. What is the most likely accompanying laboratory finding in this case?

Numerous sputum eosinophils

A 56-year-old man presents for a routine physical exam. His medical history is significant for hypertension and noninsulin-dependent diabetes for which he takes hydrochlorothiazide and metformin, respectively. His body mass index is 33 kg/m2. He states that he knows he should exercise more, but he lacks energy. Despite getting 7 - 8 hours of sleep every night, he wakes up feeling unrefreshed. In addition, his wife says he snores so loudly that sometimes she asks him to sleep on the couch. He deliberately limits his caffeine intake to 2 cups of coffee in the morning. He rarely drinks alcohol, and he denies non-prescription drug use. Based on the patient's presentation, what is the most likely diagnosis?

Obstructive sleep apnea

A 66-year-old nonsmoking male presents to the family practice clinic with complaint of a chronic cough. He notes shortness of breath on exertion. On physical exam, an increased respiratory rate with shallow breathing is noted. Dry crackles are auscultated bilaterally over the lungs. No clubbing or cyanosis is noted. The remainder of exam and vitals are normal. This patient has the following test results: Pulse oximetry: Slightly hypoxic Chest x-ray: Small opacities Pulmonary function tests (PFTs): Restrictive pattern In evaluation of this patient's chronic cough, which of the following aspects of his history are most helpful in distinguishing idiopathic pulmonary fibrosis from the various types of pneumoconioses?

Occupational exposure history

A 36-year-old African woman presents for an evaluation of a nonproductive cough, malaise, mild fever, and mild dyspnea. A chest radiograph demonstrates a right hilar mass and right paratracheal lymphadenopathy. A respiratory physician is consulted and performs a biopsy during bronchoscopy. The report reveals that the mass is a non-caseating granuloma. What medication do you prescribe?

Oral Corticosteroids

A 72-year-old man is evaluated at the bedside following hospital admission for a 1-year history of progressive dyspnea, nonproductive cough, weight loss, low-grade fevers, fatigue, and myalgias. His past medical history is remarkable for atrial fibrillation, for which he takes amiodarone, hypercholesterolemia, and recurrent urinary tract infections, for which his urologist prescribed nitrofurantoin on a chronic, prophylactic basis. He denies any cigarette use, history of murmurs or coronary artery disease, chills, fatigue, rhinitis, otalgia, chest pain, wheezing, hemoptysis, syncope, abdominal pain, rashes, peripheral edema, diaphoresis, arthralgias, vomiting, or urinary complaints. Arterial blood gas analysis demonstrated a PaO2 of 50 mmHg and pulse oximetry of SpO2 of 87%; bedside echocardiogram and electrocardiograms are unremarkable for abnormalities; a chest x-ray revealed peripheral reticular opacities at the lung bases and a generalized honeycombing pattern (idiopathic pulmonary fibrosis (IPF)). What treatment is the most beneficial in the management of this patient?

Oxygen

A 69-year-old man has a past medical history of obstructive sleep apnea, hypertension, and COPD; he presents with dyspnea on exertion that has been slowly progressive over the course of the last year. He notes impairment in climbing stairs and walking short distances. His review of systems is positive for fatigue, palpitations, intermittent retrosternal chest pain, swelling of his lower extremities, dizziness, and "feeling faint"; his associated symptoms are also known to occur upon exertion. His cardiac exam is remarkable for an oxygen saturation of 90%, accessory muscle usage, an increased pulmonic component of the second heart sound (P2), wide, inspiratory splitting of S2 over the cardiac apex, right-sided S3 and S4 gallops, a left parasternal lift, a loud diastolic murmur that increases with inspiration and diminishes with the Valsalva maneuver, prominent "A" waves in jugular venous pulsations, and increased JVD. He has an enlarged liver with hepatojugular reflux, peripheral edema, and ascites. A bedside EKG and chest x-ray were performed, which revealed the results in the image (The electrocardiogram demonstrates right axis deviation, right ventricular hypertrophy, and right atrial enlargement. The chest x-ray indicates cardiac enlargement, with prominence of the pulmonary artery, right atrium, and right ventricle.). What is the most appropriate intervention in the management of this patient?

Oxygen therapy

A 55-year-old man presents with a COPD exacerbation that is being managed with a ventilator. The patient's blood pressure drops and the ventilator alarm goes off. The only medication being administered is amlodipine via nasogastric tube. This patient is afebrile, even though the hospital has been having problems with pseudomonas infection in ventilated patients. On examination, there is a middle-aged orally-intubated man with temperature 99.4° F, pulse 145/min, and BP 62/34 mm Hg; he breathes above the ventilator at a rate of 36 cycles/min. His breathing is shallow, and there are diminished breath sounds in the right hemithorax. What is the most appropriate next step?

Perform needle thoracotomy and chest tube placement

A 21-year-old woman presents with a severe headache; it is accompanied by sweating and palpitations. She was seen by her primary care physician earlier in the day, and he admitted her from the office when her blood pressure was recorded several times at 210/98 mm Hg. The patient has no known medical or surgical history, and she is currently on a daily multivitamin. She states that she has been having her symptoms episodically over the last 2 weeks, but this is the first time she has been seen by a physician. The patient is found to have elevated urinary and plasma metanephrines and catecholamines. What antihypertensive agent should be used for this patient?

Phenoxybenzamine

A 72-year-old man is evaluated at the bedside following hospital admission for a 1-year history of progressive dyspnea, weight loss, low-grade fevers, fatigue, and myalgias. His past medical history is remarkable for atrial fibrillation, for which he takes amiodarone, hypercholesterolemia, and recurrent urinary tract infections, for which his urologist prescribed nitrofurantoin on a chronic, prophylactic basis. He denies any cigarette use, history of murmurs or coronary artery disease, chest pain, wheezing, hemoptysis, syncope, abdominal pain, rashes, peripheral edema, diaphoresis, or vomiting. Bedside echocardiogram and electrocardiograms are unremarkable for abnormalities. A chest x-ray revealed peripheral reticular opacities at the lung bases and a generalized honeycombing pattern (refer to image). Which of these statements regarding the clinical presentation of this patient is correct?

Physical exam findings of pulmonary hypertension commonly occur.

A 55-year-old woman was diagnosed with small cell cancer of the lung 2 months ago; she now presents with increasing dysphagia, respiratory difficulties, and weakness of the upper limb. Her vital signs are pulse 85/min, BP 120/90 mm Hg, resp. 12/min, and temp. 37.7° C. On examination, she has ptosis of both eyes, and she reports diplopia. Her pupillary responses are normal. The strength in the muscles of her arm on testing is 2/5; on repeated testing, the strength improves to 4/5. Sensation is intact in both upper limbs. Of the following treatments, what would be most effective in treating the patient's symptoms?

Plasmapheresis

A 2-year-old girl presents with cough, fever, decreased oral intake, and irritability for a few days. Prior to presentation, she had a 1-week history of an upper respiratory infection. On exam, she is febrile to 102°, has a respiratory rate of 28, a pulse of 110, and blood pressure of 88/56. She appears tired, but she is responsive, interactive, and in no acute respiratory distress. On lung exam, she has decreased breath sounds, dullness to percussion, and decreased tactile fremitus on the right side. What is the most likely diagnosis?

Pleural effusion

A 22-year-old man presents with sudden onset of shortness of breath and right-sided chest pain. Symptoms began yesterday, and he felt well prior to the onset of symptoms. He denies fever, hemoptysis, and upper respiratory symptoms. He is a 1 pack-per-day smoker; otherwise, he has a noncontributory past medical history. On physical exam, the patient is in mild respiratory distress; he has a slightly elevated heart rate, respiratory rate, and blood pressure. He is normotensive. His trachea appears deviated to the left. On pulmonary exam, breath sounds are diminished on the right. Hyperresonance is noted on percussion of the right chest compared to left. Other than tachycardia, his cardiovascular exam is normal. What test finding is most diagnostic for your suspected diagnosis of this patient?

Pleural line on chest X-ray (CXR)

A 22-year-old man presents with a 3-day history of sharp, localized, intermittent back pain on the right side. He denies any physical trauma to the area. He states the pain intensifies with deep breathing, sneezing, and coughing. He is also experiencing a concurrent viral respiratory illness, for which he has not sought treatment. He denies any PMH other than typical childhood illnesses. His vitals are: BP: 125/76 mm Hg, pulse: 85/min, Temp: 97.8 F. Respirations: 16/min Height: 67 inches, Weight: 170 lb. Lung exam reveals occasional coarse rhonchi throughout all lung fields, without wheezes or rales. There is no increase or decrease in tactile fremitus in any lung fields, egophony is not present, and diaphragmatic excursion is equal bilaterally. Localized tenderness in the right back at the level of ribs 7, 8, and 9 is present with deep breaths and coughing, but tenderness is not elicited with palpation of the area. A skin exam reveals no rashes or other abnormal findings. The remainder of the physical examination does not demonstrate any other abnormal findings. What is the most likely diagnosis?

Pleurisy

You are examining a 6-month-old male infant who has not received any immunizations since birth. He was born at home, and he has suffered respiratory tract infections and chronic diarrhea since birth. His parents are not related. 2 of his older brothers died of pneumonia at the age of 8 and 12 months, respectively; however, his 4-year-old sister is healthy. His parents moved to United States from an underdeveloped country 3 months ago. They are worried because 1 of their neighbors' children, who is 6 years of age, developed a pruritic skin rash and fever 2 days ago; another child, who is 6 months of age, is in the hospital because of severe vomiting and diarrhea. The parents of your patient are worried. On examination, your patient's vitals are normal; his weight is <5th percentile, his height is at the 5th percentile, and his head circumference is at the 50th percentile. Eczematous skin rash and mucocutaneous changes suggestive to a fungal infection are noted. His thyroid, heart, lungs, abdomen, genitalia, and nervous system are within normal limits; you are not able to palpate lymph nodes, and there are no signs of hepatosplenomegaly. What vaccine should you recommend?

Pneumococcal vaccine (PCV13)

A 24-year-old man presents with a 1-week history of shortness of breath and a nonproductive cough. On physical exam, he is tachycardic, tachypneic, and febrile. He has lost weight without a change in dietary habits. Auscultation of his chest reveals bibasilar crackles. A chest x-ray is ordered and demonstrates diffuse interstitial infiltrates. An arterial blood gas shows moderate hypoxemia. A metabolic panel is ordered, and the only abnormality is an isolated elevated lactate dehydrogenase (LDH) enzyme. What is the most likely diagnosis?

Pneumocystis jiroveci

A 30-year-old immunocompromised patient presents with a 2-week history of breathlessness and a nonproductive, dry cough. The patient is afebrile, pulse is 100, and BP is 110/70 mm Hg. On auscultation, scattered rales all over the chest are heard. A chest X-ray shows diffuse air space and interstitial shadowing in both lungs. The shadowing is more prevalent in the apical region. What is the most likely diagnosis?

Pneumocystis pneumoniae

A 20-year-old man presents due to developing sudden onset of shortness of breath. He is experiencing chest pain that worsens with inspiration. He has no history of pulmonary disease, but he does smoke half a pack of cigarettes daily. Vital signs reveal blood pressure 130/82, heart rate 140, respirations 28, and temperature of 98.2 degrees Fahrenheit. Heart sounds are normal with no murmurs, gallops or rubs. Lung examination reveals absent breath sounds and hyperresonance to percussion on the left side. What is the most likely diagnosis?

Pneumothorax

A 22-year-old man presents with a sudden onset of shortness of breath and right-sided chest pain. Symptoms began yesterday, and he felt well prior to the onset of symptoms. He denies fever, hemoptysis, and upper respiratory symptoms. He is a 1 pack-per-day smoker; otherwise, he has a noncontributory past medical history. On physical exam, the patient is in mild respiratory distress, with a slightly elevated heart rate, respiratory rate, and blood pressure. He is normotensive. His trachea appears deviated to the left. On pulmonary exam, breath sounds are diminished on the right. Hyperresonance is noted on percussion of the right chest compared to the left. Other than tachycardia, his cardiovascular exam is normal. A chest x-ray is obtained, and a pleural line is visible. What is the most likely diagnosis?

Pneumothorax

A 5-year-old girl has paroxysms of cough that increase in severity and duration. Some coughing episodes are followed by a whooplike, high-pitched inspiratory noise, and vomiting has also occurred after paroxysms. What laboratory test could lead to the earliest confirmation of the likely diagnosis?

Polymerase Chain Reaction assay and antigen detection

A 72-year-old man presents with an 8-month history of progressive dyspnea, which has been accompanied by a dry and persistent hacking cough. While the dyspnea now occurs at rest, he denies fever, chills, palpitations, chest pain, or peripheral edema. He states that he has worked for many years at a local chemical plant. His physical exam is remarkable for digital cyanosis and clubbing, while his pulmonary exam reveals diffuse fine, dry inspiratory crackles. His cardiac exam was positive for a prominent pulmonary valve closure sound (P2) and an elevated jugular venous pressure of 6 cm. A chest x-ray noted small lung volumes, with increased densities in the lung periphery and a honeycombing pattern; pulmonary function testing measured reductions in TLC, FEV1, and FVC with a preserved FEV1/FVC ratio. What is the best treatment for this patient at this time?

Prednisone

A 50-year-old man with no significant past medical history presents for a follow-up. He notes that, over the past week, he has developed increased nonproductive cough and shortness of breath. He has had a chronically-progressive cough and shortness of breath upon exertion over the past year. His occupational history reveals coal mining for 25 years. He denies any smoking, alcohol consumption, travel history, or sick contacts. He also denies fever, chills, sore throat, otalgia, chest or abdominal pain, peripheral edema, rashes, or pruritus. A chest radiograph was performed which revealed the following imaging (The X-ray demonstrates miliary mottling with perihilar and mediastinal lymphadenopathy showing 'egg shell calcification'). What treatment would be most beneficial in this case?

Prednisone (Deltasone)

A 28-year-old Caucasian woman presents with worsening shortness of breath with activity. The patient states that, over the last 4 weeks, it has become increasingly difficulty for her to walk up a flight of stairs without stopping after a few steps to rest. Currently, she cannot even walk across a room without feeling out of breath. She admits to significantly increased fatigue and dizziness as well as shortness of breath. The patient has a history of systemic lupus erythematosus; she was diagnosed with the condition about 2 years ago. Physical examination reveals a young woman who appears to be her stated age, but she is in a mild amount of distress due to being short of breath while just sitting on the examination table. Accessory muscle use is present. The patient's skin has a faint bluish hue to it; her lips are also a shade of blue. Ankles reveal 2+ pitting edema. What is the most likely diagnosis?

Pulmonary hypertension

A 52-year-old man presents to discuss the results of his recent lung biopsy. You saw him 3 weeks earlier due to his experiencing dyspnea. He has no other significant past medical history. His chest X-ray reveals a 3-centimeter (diameter) mass in the right upper lobe near the hilum and slight hilar adenopathy. A CT scan of his chest, abdomen, pelvis, and head reveals only the mass seen on X-ray. His blood counts and chemistries are normal. He underwent bronchoscopic biopsy of the lesion, which reveals small cell lung cancer. What is the most appropriate course of treatment?

Radiation and chemotherapy

A 40-year-old man presents in January with a 4-day history of fevers, chills, myalgias, headache, productive cough, and mild sinus congestion. He has no significant past medical history. His physical examination reveals a temperature of 101.7° F, pulse 96/min, respiratory rate 20/min, and blood pressure of 128/80 mm Hg. There is mild maxillary sinus tenderness. His oral cavity and oropharynx are clear. His tympanic membranes are pearly gray, with normal light reflex. His chest is clear to auscultation; cardiac exam is unremarkable. What is the best course of therapy for this patient?

Recommend bed rest, analgesics, and nasal decongestants

A 3-day-old male neonate is seen in the nursery due to failure to pass meconium and 2 episodes of vomiting. Prenatal and perinatal histories are unremarkable. Family history reveals a brother and sister with severe asthma. Examination reveals a moderately distended abdomen without signs of tenderness. A barium enema reveals meconium ileus with distal narrowing and proximal dilatation of the colon. Meconium is passed during the procedure, relieving the distention. What study/studies should be ordered?

Rectal manometry, rectal biopsy, sweat chloride

A 37-year-old man is admitted to the hospital after being diagnosed with tuberculosis. Before treatment is started, you inform him about the most common ways of transmitting the tubercle bacillus, as well as the medications used: isoniazid, rifampin, and pyrazinamide. What is a common side effect of rifampin therapy?

Reddish body fluids

A 68-year-old man with a history of cirrhosis presents due to weight gain, increased girth, and shortness of breath. He denies fever or chills, cough, melena, hematemesis, hemoptysis, and confusion. He takes furosemide on a daily basis, and he has not missed any doses. On physical exam, his blood pressure is 120/80; pulse is 78, and respiratory rate is 18. He is alert and cooperative. Heart exam: regular rate and rhythm without murmurs. Lung exam: decreased breath sounds on right side. Abdominal exam: positive fluid wave, no tenderness to palpation. There is no hepatosplenomegaly appreciated. Chest X-ray shows a right-sided pleural effusion; it is free flowing, as is evidenced on a decubitus film. What is the etiology of his pleural effusion?

Reduced intravascular oncotic pressure

A 58-year-old woman presents with a 2-hour history of acute-onset severe left-sided pleuritic chest pain. Her past medical history is significant for hypertension, hyperlipidemia, and breast cancer. The pain is associated with feelings of anxiety, hemoptysis, shortness of breath, and nausea. She "feels warm", but she denies chills, palpitations, wheezing, cough, edema, vomiting, abdominal pain, abnormal bowel habits, and dietary intolerances. She admits to a 30 pack-year smoking history, but she denies drug or alcohol use. Upon physical exam, she is found to be febrile, hypotensive, tachypneic, diaphoretic, and in acute painful distress. There is perioral cyanosis and a pleural friction rub to the left lung fields; the remainder of the exam is normal. What is an expected diagnostic test result for this patient?

Respiratory alkalosis on arterial blood gas analysis

A newborn infant is delivered at 30 weeks since the last menstrual period. At birth, the infant weighs 1,500gm, but otherwise appears normal. Soon after birth, the infant becomes cyanotic and breathes with a grunting noise. Chest X-rays reveal dense lungs with significant atelectasis but no cardiovascular abnormalities. What is the most likely diagnosis?

Respiratory distress syndrome

A 5-month-old infant presents with severe respiratory symptoms that include apnea and asphyxia; she is admitted to the hospital. 3 days before her admission, she developed rhinitis and a cough with wheezing. The mother became concerned when the child became lethargic. The physical examination reveals an agitated child with a persistent cough. She has a body temperature of 39°C, pulse of 190 BPM with a noted tachycardia, respirations of 76/min, and a blood pressure of 89/58 mm Hg. Her throat is clear, and auscultations of her lungs are significant for expiratory wheezing and rhonchi. Chest radiographs are positive for interstitial infiltrates and hyperexpansion. Blood gases reveal a relative hypoxemia and acidosis. The child is placed in isolation and subsequently intubated. A nasopharyngeal aspirate is sent to the laboratory for a stat rapid diagnostic test; it is positive. The child is treated with ribavirin. Based on the clinical presentation and treatment, what is the most likely diagnosis?

Respiratory syncytial virus

Early in the morning on a cold winter day, a 5-month-old infant presents with a 2-day history of severe cough, fast breathing, and fever. According to its mother, the infant is gradually getting worse and has been in close contact with other children in the neighborhood. Some of the neighborhood children were coughing when they visited the patient. The mother does not describe any paroxysmal cough. The infant's immunizations are up to date. On examination, the infant is pyrexial, tachypneic, dehydrated, and he has nasal flaring with wheezing. You admit the infant for oxygen therapy and do some blood tests to help your diagnosis. Blood test results are as follows: Leukocyte count: 6,500/mm3 Differential count: Segmented neutrophils 2 Lymphocytes 68% Eosinophils 1% Basophilic 1% Hemoglobin, blood 13.0g/dl What is the most likely causative agent?

Respiratory syncytial virus

An 18-month-old boy presents with a 2-day history of runny nose, slight cough, and low-grade fever. Over the last 24 hours, however, his condition has worsened; the child is tachypneic on exam. He is wheezing, and his breaths are associated with nasal flaring and chest retraction. What is the most common etiology of this patient's condition?

Respiratory syncytial virus (RSV)

A 69-year-old man with a 50-pack/year smoking history, COPD for 12 years, and a myocardial infarction 2 years ago has been experiencing increased exertional dyspnea for 4 months. There is associated easy fatigability, exertional chest discomfort, and lightheadedness. He denied fever, chills, palpitations, cough, wheezing, abdominal pain, nausea, vomiting, and diarrhea. Physical exam findings were remarkable for a right ventricular heave, widely split S2 with an accentuated pulmonic component, a pulmonary ejection click, an S3 and 1+ pitting edema to the bilateral lower extremities. There was also evidence of a 5 cm jugular vein distention. What diagnostic test results would be expected in this patient?

Right axis deviation, R wave greater than S wave in V1, and peaked p-waves on EKG

A New Orleans resident invites his friend to partake in a Mardi Gras tradition: the King cake. Baked into the sweet cake is a small trinket, which is shaped like a baby. The tradition is as follows: whoever gets the piece of cake with the baby in it has to buy the king cake the following year. Unfortunately, his friend accidentally bites into the trinket and inhales a small fragment of it (about 1 cm x 1 cm) while laughing. Where is the trinket most likely to lodge?

Right main bronchus

A 32-year-old African-American woman, with no significant past medical history, has been referred to a pulmonologist; she presents with a 2-month history of progressive dyspnea. She notes associated low-grade fever, malaise, joint pain, and swollen neck glands. She denies a history of travel, cigarette smoking, drug use, or sexually-transmitted diseases (she has not been sexually active in the past year). All other reviews of systems are negative. Her physical exam reveals tender, nodular formations on her anterior lower extremities, parotid enlargement, hepatosplenomegaly, and cervical lymphadenopathy. Her vital signs, heart, and lungs are unremarkable. Diagnostic testing reveals leukopenia, increased ESR, hypercalcemia, hypercalciuria, elevations of serum ACE levels, and bilateral hilar adenopathy with diffuse reticular infiltrates. ANCA, ANA, and rheumatoid factor tests are negative. Histological assessment confirms the presence of noncaseating granulomas. What is the most likely diagnosis?

Sarcoidosis

A 45-year-old woman presents with insidious onset of increasing dyspnea. A chest X-ray revealed nodular infiltrates and marked hilar lymphadenopathy. The transbronchial biopsy demonstrated noncaseating granulomas. What is the most likely diagnosis?

Sarcoidosis

A 3-year-old boy presents with a 4-hour history of respiratory distress, dysphagia, and fever. On examination, temperature is 104.3°F, pulse 150, and respirations 32 and shallow. Marked inspiratory stridor with an open-mouth appearance and sialorrhea is present. The initial step in management is:

Securing the airway

A 65-year-old retired man worked for 30 years at a mineral plant that processes diatomaceous earth. The area surrounding the plant is pretty; however, the residents have long complained that the plant produces a white dust that has coated the terrain. Over time, the man develops difficulty breathing. Finally, he sees his family doctor. He is a nonsmoker, and he denies any joint pain. Pulmonary tests show a mixed obstructive/restrictive pattern. A CBC with differential is done, and the results are all within normal limits. X-rays are done; they show eggshell calcifications of the hilar nodes. X-rays also show nodules in the upper lung fields. A biopsy of the lung shows doubly refractile particles (when viewed with polarized light). What is the most likely diagnosis?

Silicosis

A 55-year-old man presents with a nonproductive cough that has worsened over the last 2 months. He recently noted the appearance of blood-streaked sputum. Sputum cytology reveals the small cluster of hyperchromatic pleomorphic cells with very scanty cytoplasm. What is the predisposing factor to the development of these findings?

Smoking

A 69-year-old man with a past medical history of obstructive sleep apnea, hypertension, and COPD presents to his local medical office with dyspnea on exertion that has been slowly progressive over the course of the last year. He notes impairment in climbing stairs and walking short distances. His review of systems is positive for fatigue, palpitations, intermittent retrosternal chest pain, swelling of his lower extremities, dizziness, and "feeling faint." His associated symptoms are also known to occur upon exertion. His cardiac exam is remarkable for an oxygen saturation of 90%, accessory muscle usage, an increased pulmonic component of the second heart sound (P2), wide, inspiratory splitting of S2 over the cardiac apex, right-sided S3 and S4 gallops, a left parasternal lift, a loud diastolic murmur that increases with inspiration and diminishes with the Valsalva maneuver, prominent "A" waves in jugular venous pulsations, and increased JVD. He has an enlarged liver with hepatojugular reflux, peripheral edema, and ascites. A bedside EKG and chest x-ray were performed, which revealed the following image (The electrocardiogram demonstrates right axis deviation, right ventricular hypertrophy, and right atrial enlargement. The chest x-ray indicates cardiac enlargement, with prominence of the pulmonary artery, right atrium, and right ventricle). What is expected to be the most beneficial intervention for this patient?

Spironolactone

A thin 26-year-old woman presents with sudden onset of dyspnea, non-productive cough, and vague chest pain radiating to the left shoulder. She is a soccer player, and her symptoms initially occurred 24 hours ago during her usual 5-mile training run. PMH is unremarkable. EKG shows sinus tachycardia, 105 bpm. Respiratory rate equals 30 breaths per minute. Decreased breath sounds and hyperresonance are noted on the left thorax; otherwise, the physical exam is unremarkable. What is the most likely diagnosis?

Spontaneous pneumothorax

A 55-year-old man, with a 30-pack-year history of cigarette smoking, presents with a 3-month history of cough productive of blood and sputum. The patient admits a weight loss of 25 pounds over the past year. The patient also notes diffuse 'bone pain', abdominal pain, polydipsia, polyuria, and anxiety over the past month; all are unusual symptoms for him. The patient denies recent travel. The patient was treated in the emergency room twice over the past 2 months for nephrolithiasis. On chest radiography, a 3 cm cavitary lesion with an air-fluid level is noted centrally in the left upper lung field; it is considered suspicious for a pulmonary abscess. The patient also exhibits clubbing on physical examination. The patient's PPD is negative, and there is no history of anti-tuberculosis medication use in this patient's history. The patient's laboratory values are as follows: Calcium: 13.2 (normal: 8.5 - 10.8 mg/dL) Phosphorus: 1.8 (normal: 2.5 - 4.5 mg/dL) Urine cyclic adenosine monophosphate (cAMP): 9.2nmol/mL (normal: 1.6 - 6.2 nmol/mL) What is the most likely diagnosis?

Squamous cell carcinoma

A 55-year-old man presents with a 4-hour history of severe left-sided chest pain. His wife tells you that they went out to dinner earlier in the evening, but the patient felt ill shortly after returning home. He had several episodes of vomiting, followed by severe pain. On exam he appears ill; he is cool and diaphoretic. Blood pressure is 90/60, heart rate is 126, respiratory rate is 32, and temperature is 39.8o C. There is no JVD. Cardiac exam is tachycardic, but there is no murmur. Lung sounds are decreased in the left base, with dullness to percussion. The abdominal exam reveals tenderness, but there is no guarding or rebound. Laboratory studies reveal a white blood cell count of 14,000, a hemoglobin of 14 gm/dL, and a hematocrit of 44 gm/dL. EKG reveals a sinus tachycardia without any Q waves. Chest X-ray reveals a left pleural effusion. The described case scenario represents a patient with Boerhaave's syndrome, which is a spontaneous esophageal rupture after forceful vomiting. Once the diagnosis is confirmed, what is the optimal definitive treatment?

Surgical exploration

You are evaluating a 6-year-old child with a history of recurrent pneumonias. The child has a history of frequent ear infections and often gets sick due to pneumonias, requiring the frequent use of antibiotics. He has a chronic wet cough that is present most of the time, and he also has failure to thrive and poor weight gain. As part of the diagnostic work up, what would be the most useful when differentiating between Cystic Fibrosis (CF) and Primary Ciliary Dyskinesia (PCD) in this patient?

Sweat chloride level of >60mmol/L

A 2-week-old neonate has been reported to have an elevated Immunoreactive Trypsinogen (IRT) level on her newborn screening sample that was sent from the newborn nursery after birth. The state newborn screening lab also ran additional tests to screen for the 40 most common mutations that are known to cause Cystic Fibrosis (CF). The result from that mutation screening was negative. What should be the next step in the evaluation of this positive newborn screening result for Cystic Fibrosis?

Sweat chloride test

A 3-year-old girl presents for a respiratory check due to her 3rd episode of pneumonia this year. She has moderate crackles in the lower right lung base, but she is afebrile and breathing comfortably. Past medical history is significant for a few episodes of pneumonia each winter since birth. She has always been small for her age, but her mother says she has a healthy appetite. Her parents and brother are of medium stature. She takes no medication other than the antibiotic that was prescribed 2 days ago. What test should be considered once she has recovered completely from pneumonia?

Sweat chloride test

A 57-year-old woman presents to the hospital complaining of a 2-week history of feeling weak. She is an artist who has been chain smoking for 30 years. On examination, she is kempt and wasted but not dehydrated. Her BP is 116/70 mm Hg. Laboratory investigations reveal plasma sodium of 122 mEq/L with a high urine osmolality. A chest radiograph reveals a rounded opacity in the right lung field. What is the most likely diagnosis?

Syndrome of inappropriate antidiuretic hormone secretion (SIADH)

A 1-month-old formerly premature male infant with BPD remains intubated and monitored in the NICU. He has been doing relatively well and is being gradually weaned from the respirator. Suddenly, his O2 saturations and heart rate plummet, and he becomes very dusky. On quick exam, there are decreased breath sounds on the right with an asymmetric chest rise. What is the most likely explanation for his sudden respiratory and clinical change?

Tension pneumothorax

A 3-month-old male infant presents with history of noisy breathing since birth; the noise is gradually increasing. There is no history of fever, cough, or running nose. Physical examination reveals a low-pitched, expiratory wheeze; it is loudest over the trachea. Wheezing increases during crying, feeding, and when the infant is laid in supine position. There is no cyanosis, subcostal or intercostal retraction, or hoarseness of voice. Wheezing has not shown any response to bronchodilators. What is the most likely diagnosis?

Tracheomalacia

A 72-year-old man presents due to worsening shortness of breath, orthopnea, and chest pain; symptoms have been occurring for the past few weeks. The patient admits to some chronic heart problems as well as fatigue, dyspnea, and a non-productive cough; however, he feels like symptoms have worsened recently. He denies fever, chills, and a productive cough. On physical exam, the man has mildly increased respiratory effort, but he does not appear in distress. He is barrel-chested. His breath sounds are diminished bilaterally, with dullness to percussion over the right and left lower lungs. No pleural friction rub is noted. On cardiovascular exam, an S3 gallop and mild tachycardia (110 bpm) are noted. Clubbing of the fingers, dependent edema in the lower extremities, and jugular venous distention are also noted. His cardiac enzymes and electrocardiogram demonstrate no acute cardiac pathology. Pleural fluid and cardiomegaly are found on the chest X-ray, and a thoracentesis is performed. The pleural fluid is generally clear in color, testing negative for chylomicrons and triglycerides. It has low levels of red blood cells, white blood cells, protein, and lactate dehydrogenase (LDH). What is the likely underlying mechanism for the pleural effusion in this patient?

Transudates from increased hydrostatic pressure or decreased oncotic pressure

A 68-year-old nonsmoking man presents to your pulmonology practice for long-standing dyspnea and nonproductive cough. The patient has had the cough and progressively worsening dyspnea for about 1.5 years, with no current exacerbation. He denies fevers, chills, night sweats, and any other symptoms. He denies unusual travel, hobbies, or occupational exposures. He does, however, endorse relatively frequent reflux symptoms. The rest of his review of systems is negative. He has had a workup through his family practice and was then sent to the cardiologist, who ruled out cardiovascular causes of his dyspnea. He has been given trials of various antibiotics, inhalers, and steroids, all without improvement in symptoms, despite good compliance. He currently takes no medications. Several tests have been performed, and results are shown in the table. Blood count, metabolic panel, HIV, and autoimmune markers: Normal High-resolution computed tomography (HSCT) scan: Few reticular opacities Pulmonary function test (PFT): Restrictive impairment and reduced perfusion of carbon monoxide Physical exam is significant for fine inspiratory bibasilar crackles and clubbing in the fingers. An occasional dry cough is noted. Which of the following interventions is MOST likely to prevent complications in this patient?

Treatment of gastroesophageal reflux disease (GERD)

A 45-year-old man presents with significant weight loss (10 pounds in 4 months), cough with hemoptysis, and pleuritic chest pain. The chest X-ray shows ill-defined opacities in both the lungs; the opacities have a reticulonodular pattern. A transbronchial biopsy is performed, and it microscopically shows a few epithelioid cells with necrotic debris. What is the most likely diagnosis?

Tuberculosis

A 37-year-old man with an unremarkable past medical history presents during a cold winter's day with a 10-day history of acute onset of productive cough with a moderate amount of yellow sputum. There is associated fever, shortness of breath, and malaise. He denies recent travel, sick contacts, occupational exposure, and a history of smoking or alcohol use. He denies arthralgias, chills, wheezing, abdominal pain, nausea, vomiting, diarrhea, edema, or rashes. His physical exam is remarkable for fever, tachypnea, reduced fremitus, dullness to percussion, and basilar crackles in the right lower lung field. What additional presentation finding would be most likely in this patient?

Unilateral, sharp inspiratory chest pain and thoracic friction rub

A 9-month-old boy presents with a 3-day history of low grade fever and rhinorrhea, followed by 1 day of barking cough and difficulty breathing. The child's cough has been progressively getting worse. He has had a low-grade fever for 2 days. He is eating poorly, but there is no emesis or diarrhea. There is no significant past medical history and all his vaccinations are up-to-date. No one else in the household is currently ill. The child lives with his mother and a 2-year-old sister, and he attends day care. On physical exam, he has a temperature of 38.5 degrees Celsius, an elevated respiratory rate of 48, and normal heart rate and blood pressure. On lung exam, there are subcostal inspiratory muscle retractions; on auscultation, there is a high-pitched inspiratory stridor. What is causing the patient's symptoms?

Viral infection of the larynx and trachea

A 40-year-old woman undergoing chemotherapy develops fevers and respiratory symptoms. A chest radiograph is ordered, and it reveals bilateral fluffy pulmonary infiltrates. A bronchoscopy with biopsy is performed, and the specimen is found to contain septate hyphae with acute-single branching. What treatment should the patient receive?

Voriconazole


Related study sets

Chem 175 - Chapter 16 (cont.) Exam 4

View Set

Chapter 3: Making Apps More Interactive Through Data Input

View Set

Unit 2, Assign. 2: Skills and Aptitude

View Set